Mate in 2 - how many solutions?

Mate in 2 - how many solutions?

Posers and Puzzles

Cookies help us deliver our Services. By using our Services or clicking I agree, you agree to our use of cookies. Learn More.

S
Caninus Interruptus

2014.05.01

Joined
11 Apr 07
Moves
92274
11 Feb 09


#2

A

Joined
02 Mar 06
Moves
17881
12 Feb 09
1 edit

i think there's only one... 1.Rd1..[insert any black move here] 2.Rd8#

the other variations fail: 1. Rxa4 .. e3! (2.Ra8+ .. Bxa8), 1. Rb1..Nb2! (or Nb6!), 1.Rc1..Nc3! (or Nc5!). and if Ra2, Ra3, or any other white move, black can just push his bishop to g2 or f3, or move a pawn, so long as he retains his knight at a4 he avoids mate in 2.

edit: and note that black has no mating solutions, so even though it's not explicitly stated that it's a white mate in 2, it does not present any issues

S
Caninus Interruptus

2014.05.01

Joined
11 Apr 07
Moves
92274
12 Feb 09

Originally posted by Aetherael
i think there's only one... [b]1.Rd1..[insert any black move here] 2.Rd8#

the other variations fail: 1. Rxa4 .. e3! (2.Ra8+ .. Bxa8), 1. Rb1..Nb2! (or Nb6!), 1.Rc1..Nc3! (or Nc5!). and if Ra2, Ra3, or any other white move, black can just push his bishop to g2 or f3, or move a pawn, so long as he retains his knight at a4 he avoids mate in ...[text shortened]... though it's not explicitly stated that it's a white mate in 2, it does not present any issues[/b]
Is that your final answer?

F

Joined
11 Nov 05
Moves
43938
12 Feb 09

Originally posted by Aetherael
i think there's only one... [b]1.Rd1..[insert any black move here] 2.Rd8#
[/b]
How about 1. 0-0-0, and then 2. Rd8#

A

Joined
02 Mar 06
Moves
17881
12 Feb 09

Originally posted by FabianFnas
How about 1. 0-0-0, and then 2. Rd8#
ooo sneaky monkey swissgambit! nice catch FabianFnas. i don't see any evidence to refute white's ability to castle on the queenside so that looks like a solution.

h

Joined
25 Apr 06
Moves
5939
12 Feb 09
1 edit

Originally posted by SwissGambit
How many solutions?
One [1. Rd1 ~ 2. Rd8#]. bBf8 was captured at home; bBh6 comes from a promoted pawn on a1 (from a7), so the a1 Rook must have moved; if the a7 pawn promoted on c1, that would have required 15 captures.

A

Joined
02 Mar 06
Moves
17881
12 Feb 09
1 edit

Originally posted by heinzkat
One [1. Rd1 ~ 2. Rd8#]. bBf8 was captured at home; bBh6 comes from a promoted pawn on a1 (from a7), so the a1 Rook must have moved; if the a7 pawn promoted on c1, that would have required 15 captures.
maybe i'm missing something but what stops the black bishop from surviving through the e-pawn capturing to f6, the h-pawn capturing to g6, and then bB moves to a3, c1, and h6 before blacks pawn continues to g5? then i count only 13 black pawn captures...

i'm still not good at these problems though so if you have some insight it would be much appreciated! 🙂 i really like the logic behind the retro analysis games

edit: in rereading i'm not very clear in my question. why must the black DS bishop have been captured at home, rather than moving in a legal order to reach h6 within the current pawn formation? am i missing a key issue in which this order of moves is impossible?

h

Joined
25 Apr 06
Moves
5939
12 Feb 09

Originally posted by Aetherael
maybe i'm missing something but what stops the black bishop from surviving through the e-pawn capturing to f6, the h-pawn capturing to g6, and then bB moves to a3, c1, and h6 before blacks pawn continues to g5? then i count only 13 black pawn captures...

i'm still not good at these problems though so if you have some insight it would be much appreciated! 🙂 i really like the logic behind the retro analysis games
The Rook on g6 I think.

A

Joined
02 Mar 06
Moves
17881
12 Feb 09
2 edits

Originally posted by heinzkat
The Rook on g6 I think.
oh of course. very good thanks! and great problem swissgambit - i always love the subtleties of these constructions that seem obvious at first, have some sort of complicating/"oh i forgot to consider the possibility of ___"-twist, and yet are ultimately entirely concise and sound.

h

Joined
25 Apr 06
Moves
5939
12 Feb 09

Originally posted by Aetherael
oh of course. very good thanks!
Probably this is still incorrect but OK, I'm glad it sort of sounded convincing to you 🙂

A

Joined
02 Mar 06
Moves
17881
12 Feb 09

Originally posted by heinzkat
Probably this is still incorrect but OK, I'm glad it sort of sounded convincing to you 🙂
haha... well it at least contradicted my proposed move order, in that black's a-file rook would not be able to sneak in there behind both black's DS bishop AND the h-pawn capturing at g7 followed by g7-g6. and i can't think of any other move configuration that would allow the pieces to be setup that way, so am more inclined to agree that the black bishop is a result of an a1 pawn promotion, since as you said a c1 promotion would require 15 pawn captures and thus make white's rook impossible.

S
Caninus Interruptus

2014.05.01

Joined
11 Apr 07
Moves
92274
12 Feb 09
1 edit

Team SOLV'D. I will only add a supplement on why Bh6 must be promoted.


Consider this part of the structure. Which of these pieces moved last? There is only one that can even retract a move - Pf6 - and he must have come from e7. This proves conclusively that Bh6 could not have come from f8, but is promoted.

The answer is one solution - 1.0-0-0? is illegal per the reasoning here and in posts above.

S

Joined
26 Oct 08
Moves
1379
18 Feb 09
1 edit

How did Black got three pawns on the G-file, two on the f-file and one the the h file ?

U

Joined
22 Sep 06
Moves
1707
18 Feb 09

Originally posted by Sophy
How did Black got three pawns on the G-file, two on the f-file and one the the h file ?
The pawn on h2 came from c7. The pawn on g4 came from d7. The pawn on g5 came from h7 and the pawn on f6 came from e7.

S
Caninus Interruptus

2014.05.01

Joined
11 Apr 07
Moves
92274
19 Feb 09

Originally posted by Sophy
How did Black got three pawns on the G-file, two on the f-file and one the the h file ?
Black obviously captured several white pieces with pawns.